I need help with this

I Need Help With This

Answers

Answer 1

Answer:

Step-by-step explanation:

the given equation is in the form of quadratic formula i.e.ax^2+bx+c=0.

So ,here a=-1 ,b=-6 and c=-9

b^2=(-6)^2

=36

4ac=4*(-1)*(-9)

=4*9

=36

b^2-4ac=36-36

=0

number of solutions =1


Related Questions

Suppose I have an urn with 9 balls: 4 green, 3 yellow and 2 white ones. I draw a ball from the urn repeatedly with replacement. (a) Suppose I draw n times. Let X., be the number of times I saw a green ball followed by a yellow ball. Calculate the expectation Ex, (b) Let y be the number of times I drew a green ball before the first white draw. Calculate E[Y]. Can you give an intuitive explanation for your answer

Answers

Answer:

[tex]E(X_n)=\frac{2(n-1)}{27}[/tex]

[tex]E(y)=\frac{14}{9}[/tex]

Step-by-step explanation:

From the question we are told that:

Sample size n=9

Number of Green [tex]g=4[/tex]

Number of yellow [tex]y=4[/tex]

Number of white [tex]w=4[/tex]

Probability of Green Followed by yellow P(GY) ball

 [tex]P(GY)=\frac{4}{9}*\frac{3}{9}[/tex]

 [tex]P(GY)=\frac{4}{27}[/tex]

Generally the equations for when n is even is mathematically given by

 [tex]Probability of success P(S)=\frac{4}{27}[/tex]

 [tex]Probability of Failure P(F)=\frac{27-4}{27}[/tex]

 [tex]Probability of Failure P(F)=\frac{23}{27}[/tex]

Therefore

 [tex]E(X_n)=\frac{n}{2}*P[/tex]

 [tex]E(X_n)=\frac{n}{2}*\frac{4}{27}[/tex]

 [tex]E(X_n)=\frac{2n}{27}[/tex]

Generally the equations for when n is odd is mathematically given by

 [tex]\frac{n-1}{2}[/tex]

 [tex]E(X_n)=\frac{n-1}{2}*\frac{4}{27}[/tex]

 [tex]E(X_n)=\frac{2(n-1)}{27}[/tex]

b)

Probability of drawing white ball

 [tex]P(w)=\frac{2}{9}[/tex]

Therefore

 [tex]E(w)=\frac{1}{p}[/tex]

 [tex]E(w)=\frac{1}{\frac{2}{9}}[/tex]

 [tex]E(w)=\frac{9}{2}[/tex]

Therefore

 [tex]E(y)=[E(w)-1]\frac{4}{9}[/tex]

 [tex]E(y)=[\frac{9}{2}-1]\frac{4}{9}[/tex]

 [tex]E(y)=\frac{14}{9}[/tex]

Math help please urgent

Answers

Answer:

THREE AND SIX OR FOUR AND FIVE

Step-by-step explanation:

They're just opposite ends !

Step-by-step explanation:

Alternate Interior Angles: 3 and 6

Alternate Exterior Angles: 2 and 7

A person runs 1/4 mile in 1/28 hour. What is the persons speed

Answers

Answer:

7

Step-by-step explanation:

speed x time = distance

? x time = distance

1/28x = 1/4

1/28x          1/4

_____ =  _____

1/28          1/28

1/4 ÷ 1/28

(reciprocal method) 28/1

1/4 × 28/1 (change the equation into multiplication)

1/4 × 28/1 = 28/4 (multiply)

(Simplify) 28/4 ---> 7/1 or 7

final answer:

The persons speed would be 7.

PLEASE HELP!! The green line below *blank*. Check all that apply.

A. is parallel to the x-axis

B. crosses the origin

C. is in the xy-plane

D. crosses the y-axis

Answers

Answer:

D. crosses the y-axis

[tex]{ \tt{crosses \: y - axis \: at \: y = 5}}[/tex]

En un sistema masa resorte (masa atada a un muelle) ¿en qué punto la masa alcanza su máxima velocidad?

Answers

Answer:

yo just search up Spanish translator

¿¡ Cuanto es "(8 x 7 + 5 x (-8)) : (-4)" ??

Answers

Answer:

-4

Step-by-step explanation:

So like ur last problem do it in order left to right.

[8x7]+5x(-8)/(-4)

56+[5x(-8)/-4

56+(-40)/-4

16/-4=(-4)

Sorry if i did in wrong order.

Answer: 4(14x-11) hop that can help you

In the diagram of a circle shown below, m AC = 89° and m BC = 153°. Determine m∠ACB.

Answers

Answer:

The measure of the angle ACB is 59°.

Step-by-step explanation:

Let be [tex]D[/tex] the location of the center of circle and [tex]r[/tex] is the radius of the figure, the triangles ACD and BCD are isosceles triangles, since [tex]AD = CD = BD = r[/tex]. And the measure of the angle C associated with each triangle are, respectively:

Triangle ACD

[tex]m\angle C = \frac{180^{\circ}-89^{\circ}}{2}[/tex]

[tex]m\angle C = 45.5^{\circ}[/tex]

Triangle BCD

[tex]m \angle C = \frac{180^{\circ}-153^{\circ}}{2}[/tex]

[tex]m\angle C = 13.5^{\circ}[/tex]

Lastly, the measure of the angle ACB is:

[tex]m\angle ACB = 45.5^{\circ} + 13.5^{\circ}[/tex]

[tex]m\angle ACB = 59^{\circ}[/tex]

The measure of the angle ACB is 59°.

Help me ASAP please

Answers

Answer:

I think its D because if it has the parentheses then that would mean that it would go right or left

Coach Peters has 12 gallons of water to fill buckets for field day. If
each bucket needs of a gallon to fill, how many buckets can he fill?

Answers

Answer:

Step-by-step explanation:

D.36

because 12 divided by 1/3 is 36

What are the more appropriate measures of center and spread for this data set?

Answers

Answer:

Better measure of spread: The Interquartile Range

Better measure of center: The median

Step-by-step explanation:

I've done the quiz and got it correct:)

Shane and Abha earned a team badge that required their team to collect no less than 2000 cans for
recycling. Abha collected 178 more cans than Shane did.
Write an inequality to determine the number of cans, S, that Shane could have collected.
What is the solution set of the inequality?

Answers

Answer:

s + 178 = Abha's Can's

Step-by-step explanation:

Well since Abha collected 178 more cans than Shane, it would be Shane's amount of cans they collected AND 178 cans on top of that.

(i didnt know if you wanted the solution of s or just the equation on how to find s)

The area of a rectangular parking lot is 3960 m²
If the length of the parking lot is 72 m, what is its width?

Answers

Answer:

3816m

Step-by-step explanation:

72 + 72= 144

8960 - 144= 3816

I’m struggling in finding the nearest cent, May you please help me

Answers

Answer:

$88.48

Step-by-step explanation:

First, we can check the formula for compound interest, which is

[tex]A = P (1 + r)^{n}[/tex] , with A representing the end amount, r representing the interest rate, and n representing the amount of times compounded. We know that the interest rate is 9% (to convert this to a decimal, we can divide it by 100, resulting in 0.09), the end amount is 700, and the amount of times compounded is however many months are in 2 years (there are 12 months per year, and 12 *2 = 24, so 24 times).

Thus, our equation is

[tex]700 = P (1 + 0.09)^{24}\\700 = P (7.911083)\\[/tex]

Divide both sides by 7.911083 to isolate P, and we get

P = 88.48 (rounded to the nearest cent)

What is the median for this set of data?
28, 32, 15, 19, 24, 26, 22, 32, 30

Answers

Step-by-step explanation:

according order 15 , 19,22, 24,26,28,30,32,32. answer is 26

Answer:

Hello There!!

Step-by-step explanation:

26 is the median as you order the numbers smallest to largest.Therefore,it is 15,19,22, 24, 26,28,30,32,32 and the amount of numbers are odd so the median is the middle number.If it was even the two middle numbers would be added together and then divided by 2.

hope this helps,have a great day!!

~Pinky~

Whats the y-intercept of -2x - y=3

Answers

Let’s first write this in the slope form: y=mx+b.
m is the slope and b is the y intercept.
-2x-y=3
y=-2x-3
So the b, or the y intercept is -3

A track coach records the number of miles that he ran per day over the month. The stem-and-leaf plot below summarizes this data.

Answers

Answer:

[tex]Mean = 6.70[/tex]

Step-by-step explanation:

Given

See attachment for plot

Required

The mean of the plot

Mean is calculated as:

[tex]Mean = \frac{\sum x}{n}[/tex]

Where

[tex]n = 20[/tex]

Using the given key to read, the plot;

We have:

[tex]Mean = \frac{3.00+3.25+4.25+5.00+..........+8.75+9.50}{20}[/tex]

[tex]Mean = \frac{134.00}{20}[/tex]

[tex]Mean = 6.70[/tex]

What is an equivalent expression for 6v + -1 ?

Answers

Answer:

5v is an equivalent expression for 6v + -1 ?

Answer:

5v is equivalent expression for 6v + -1

HELP ASAP PLEASE!!!!!

Answers

I think the answer is (6,-4)
i think it will be (4,-4)

pls help me with this math problem​

Answers

Answer:

The four rational numbers between [tex]-\frac{1}{2}[/tex] and [tex]\frac{1}{2}[/tex] can be:

[tex]-\frac{1}{3}[/tex][tex]-\frac{1}{4}[/tex][tex]\frac{1}{5}[/tex][tex]\frac{1}{7}[/tex]

Step-by-step explanation:

The definition of rational numbers is usually a number of the type a/b where b is different from zero, therefore, to choose the numbers that are between -1/2 (= -0.5) and 1/2 (= 0.5) it is enough with choose fractional numbers whose dividend is 1 and its divisor is any number greater than 2, either negative or positive, in the answer we choose options such as 1/7 or -1/3, but following the guide I give you, you could choose numbers like 1/16, -1/9 or 1/40 if you wish, always respecting the guidelines given for the dividend and the divisor.

Lines J and k intersect at point Q and h is a straight line. What is the value of X ?

Answers

Answer:

Step-by-step explanation:

Angles on a straight line add up to 180 so 180-135 equals 45. Vertically opposite angles are equal so the second angle inside the triangle equals 100. Henceforth, we can solve this equation:

x +45+100 = 180 (angles in triangle add up to 180)

x+145=180

x=35

Vertically opposite angles are equal so x will become 35.

Hope this helps!

Dois triângulos semelhantes possuem razão entre suas áreas igual a 9. Se o perímetro de um deles é 10, o perímetro do outro deve ser:

Answers

Answer:

O perímetro do outro triângulo deve ser ou de 30 unidades de comprimento, ou de 3.33 unidades de comprimento.

Step-by-step explanation:

Dois triângulos semelhantes possuem razão entre suas áreas igual a 9.

O perímetro tem grau um, enquanto a área tem grau 2. Isto implica que a razão entre os perímetros é a raiz quadrada da razão entre as áreas, então a razão entre os perímetros é de 3.

Se o perímetro de um deles é 10, o perímetro do outro deve ser:

Ou 10*3 = 30, ou [tex]\frac{10}{3} = 3.33[/tex]

O perímetro do outro triângulo deve ser ou de 30 unidades de comprimento, ou de 3.33 unidades de comprimento.

Evaluate expressions-Match each expression to another expression with the same value

Answers

Answer:

24 / 3

8 x 50

53 x 7

The vertices of the parallelogram below are given. Use matrices to find the area.

A (0, 0) B (-2, 3) C(4, -1) and D(2, 2) HELP PLEASE HURRY

Answers

Answer:

B

Step-by-step explanation:

There are 2121 students in a school. A PE teacher wants to arrange maximum possible number of students in the field such that the number of rows is equal to the number of columns. How many students are left out in this arrangement?

Answers

Answer:

5 students are left out in the arrangement.

Step-by-step explanation:

If the number of rows = the number of columns  

Then there must be an equal arrangement

Since the total number of students in the school is 2121

Then, the students that are left out in this arrangement = 2121 - (√2121 X √2121

Note the result of the square root would only consider the whole number, the digits after the decimal point signifies the remaining number that can't fit into the arrangement

so, √2121 = 46.05 (so 46 would be used)

= 2121 - (46 X 46)

= 2121 - 2116 = 5

Therefore 5 students are left out in the arrangement.

what is the value of x?​

Answers

Answer:

Step-by-step explanation:

The sum of interior angles in a triangle is equal to 180 degrees

so

70 + 35 + x = 180 add like terms

105 + x = 180 subtract 105 from both sides

x = 75

Answer:

75  degrees

Step-by-step explanation

triangles add up to 180 degrees so, 70+35= 105 degrees, 180-105= 75 degrees

The diagram shows the locations of Cody and Reese after running
different directions from a tree. How far did Cody run?

Answers

Answer: 80yd

Step-by-step explanation:

What is the product?

Answers

Answer:

[tex]21x^{7}y^{11}[/tex]

Step-by-step explanation:

Remove the parentheses and take out the constansts.

[tex](7 x 3) x^{2} x^{5} y^{3} y^{8}[/tex]

Simplify 7 x 3 to 21.

[tex]21x^{2} x^{5} y^{3} y^{8}[/tex]

Use the product rule: [tex]x^{a} x^{b} = x^{a + b}[/tex]

[tex]21x^{2 + 5}y^{3+8}[/tex]

Simplify 2+5 to 7.

[tex]21x^{7}y^{3+8}[/tex]

Simplfiy 3 + 8 to 11.

[tex]21x^{7}y^{11}[/tex]

To be considered abnormal height a 30 year old must be 1.2 feet below average or 1.2 feet above
average. If the average height is 5.1 feet, at what heights (in interval notation) would a 30 year old
be considered abnormal?
M

Answers

Answer:

The interval notation would be (-∞, 3.9 feet) ∪ (6.3 feet, ∞).

Step-by-step explanation:

Since the lower and upper bounds are given, those very bounds extend to infinity in this case since there is no limit to the height which would be considered abnormal in this question. Using the lower bounds and upper bounds to construct an interval notation, we get the answer as shown above.

Hope this helped!

Find the circumference of each circle from the given radius or diameter

Please help

Answers

Answer: 1) 113.20 yds 2) 75.40 yds 3) 44 yds

*These answers were calculated using the true value of pi, not 3.14*


Hope this helps シ

Find a, b and c for the function x^2 + 2x - 6 = 0

Answers

Remember y = ax^2 + bx + c

That means a = 1, b= 2 and c = -6
Other Questions
Solve for t.3t= -8.16 Miners quickly realized that the newly discovered Comstock Lode would require different technology than mines in California because Group of answer choices A) the ore was so pure that it was difficult to extract. B) the ore was deep underground. D) the ore was richest in the streams and rivers. C) the rock surrounding the ore was too hard for picks and hammers. Give three benefits of asexual reproduction in plants to us as humans. Atoms are attracted to each other because____A) electrons are attracted to protons because they have opposite charges B) magnets pull them together C) the protons are attracted to each other because they have the same charge D) they have some energy Act II, Scene ii opens with a series of soliloquies before Romeo and Juliet speak to each other. What is the best description of the effect these soliloquies have on the scene?The soliloquies make the entire scene feel more realistic. The soliloquies make the balcony scene more tragic.The soliloquies create a romantic, intimate mood. The soliloquies suggest that the scene is not real, but only a dream. You are on a ship at sea. You see a lighthouse in the distance. You know the lighthouse is 175 feet tall. You measure the angle of elevation to the top of the lighthouse at 5 degrees. How far are you from the bottom of the lighthouse What can countries do to prevent terrorism is equivalent to 16 Superscript three-fourths? $5 out of every $100 a worker earns goes for clothes. On $4,000 a year wages, how much does he spend on clothes? Selling food locally increases sustainable use of the land because crops can be rotated more easily. What is a cost of this practice?A. There is less CO2 emitted from transportation.B. Less water is used to irrigate the crops.C. Food spoils more quickly when it is not processed.D. More packaging is needed for transportation.( The "There is less CO2 emitted from transportation" is not correct, I've tried it twice, and have gotten it incorrect each time, please help!!! how long will they take to make 504 muffins if they will be using 4 ovens? A function is defined by f(x)= 6x+1.5. What is f(2.5)? tell me a some basic terms related bijective.1:A function of F:R --->is defined by f(x)=x-1,is f one to one an onto.2:A function F:[-2,2]-->R is defined by f(x)=x is F one to one and onto.3:A function F:[1,4]-->R is defined by f(x)=x is F one to one and onto Write the log equation as an exponential equation. You do not need to solve for x. \log_9\left(2\right)=2x-1 log 9 (2)=2x1 Discover the rate of change (slope) of these two points. (-4, 5) and (5,-4). Rosa and Micah are playing a game . Which table is the closest to showing Rosa winning approximately plays ? 3/5 of the times she A ustedes ______ gusta mucho el cereal con lecheIs it...?1. me2. te3. le4. les5. nosWhich one is it?? Please help Which of the following are clear examples that the "thaw" of the Cold War did not last? [Selectall that apply.]Russia suppressed Hungary after citizens demanded reform.Russia suppressed Poland.Russia invaded Japan.Russia suppressed Czechoslovakia after citizens demanded reform. goodie morningwhat is nitric acid? The rate at which electriccharges flow through acircuit